Calcular el valor del Límite

A mí me da 1/2, pero no es una de las posibles respuestas, a ver si con tu aclaración podría sacarlo.

Calcular el valor del

$$\lim_{n \to \infty} \frac{1+\frac{n}{n+1}+...+\frac{n}{n+(n+1)}}{n} = 0$$

A)ln 2

B)0

C)Infinito

D)1

Gracias!!

1 respuesta

Respuesta
1

Como puedes ver, no aparece la expresión de la que tengo que calcular el límite. Creo que esta pregunta la mandaste hace unos días y estuve intentando demostrarla pero no pude, creo que el límite era algo asi como

$$\lim_{n \to \infty} \left (  1+\frac{n}{n+1}+...+\frac{n}{n+n+1}\right )$$

Ya me corregirás si no estoy en lo cierto.

Si todos los sumandos fueran como el último si que tendería a 1/2, pero los anteriores son mayores, comienzan teniendo límite 1 y acaban teniendo limite 1/2, la suma es una cantidad intermedia que muy bien pudiera ser la solución A) ln 2. Lo que pasa es que en ese momento tenía mucho trabajo y no encontré en internet la fórmula de la suma de los primeros números inversos.

Si me confirmas el enunciado del problema intentaré hacer algo.

el último sumando tiene como denominador n+(n-1).

Yo lo intente por Stolz pero me sigue dando 1/2.

Espero que así te llegue bien y que te vayas desliando pronto.

Un saludo y gracias!!!

Espera, no puede ser eso, ese límite es infinito. Creo que era este

$$\lim_{n \to \infty}\frac{1+\frac{n}{n+1}+...+\frac{n}{n+n+1}}{n}$$

pero mejor confírmame que lo es.

Lo aclaré arriba...hubo conflicto a la hora de enviar...

Si, era ese segundo, si no ponemos el n debajo del todo, el límite sería infinito. El que termine en n+n-1 o n+n+1 no influye, cada sumando conveniente dividido por n tiene límite cero, entre todos valdrán algo porque son infinitos.

$$\lim_{n \to \infty} \; \frac{1}{n}+\frac{1}{n+1}+...+\frac{1}{2n-1}$$

Como te decía, he intentado encontrar una fórmula que diga cuál es la suma de los primeros n números recíprocos, pero no debe existir, no será fácil.

Lo que sí he encontrado es esto:

http://es.wikipedia.org/wiki/Número_armónico

Ahi se llama número armónico

$$H_n= \sum_{k=1}^n \frac{1}{k} = 1+\frac{1}{2}+\frac{1}{3}+...+\frac{1}{n}$$

Y luego dice que esa suma se puede calcular aproximadamente por

$$H_n \approx \int_1^n \frac{1}{x}dx = ln x|_1^n = ln \; n$$

Luego aún da una aproximación mayor diciendo

$$H_n= \gamma + ln \; n+ \frac{1}{2n}-\frac{1}{12n^2}+\frac{1}{120n^4}+O(n^{-6})$$

En realidad con la de arriba nos sobraría para lo que vamos a hacer.

$$\begin{align}&\lim_{n \to \infty} \; \frac{1}{n}+\frac{1}{n+1}+...+\frac{1}{2n-1}= \lim_{n \to \infty} H_{2n-1}- H_{n-1}=\\ &\\ &\\ &\lim_{n \to \infty}\gamma-\gamma+ ln(2n-1)-ln(n-1)+\frac{1}{2(2n-1)}- \frac{1}{2(n-1)}+...\end{align}$$

Y como decía no pongo más porque todo lo que va detrás de los ln es un infinitésimo de orden superior y tiende a cero con mayor velocidad. Si no te fías puedes comprobarlo haciendo las cuentas con todos los términos.

En resumen queda:

$$\begin{align}&= \lim_{n \to \infty} ln(2n-1)-ln(n-1)=\\ &\\ &\\ &\lim_{n \to \infty} ln \left ( \frac{2n-1}{n-1} \right )=\\ &\\ &\\ &ln \left (  lim_{n \to \infty} \frac{2n-1}{n-1} \right )=ln \; 2\end{align}$$

Y eso es todo.

MADRE MIA!

No se donde se ha podido sacar esto el profesor, ya que no lo he visto en todos los apuntes. Lo importante es que sucesiones del tipo 1/n los entiendo.

Muchas gracias!!!

Añade tu respuesta

Haz clic para o

Más respuestas relacionadas